Soportes de Poisson y campo magnético [cerrado]

Soy un estudiante de matemáticas que intenta aprender algo de física, así que lo siento si me estoy perdiendo algo simple aquí. Creo que el principal problema es la falta de experiencia con el símbolo de Levi-Cevita.

Tenemos una partícula en un campo magnético. B = × A , descrito por el hamiltoniano (tomando C = 1 ):

H = 1 2 metro ( pag mi A ( r ) ) 2 = metro 2 r ˙ 2

Me dicen que la estructura de corchetes de Poisson dice { metro r ˙ a , metro r ˙ b } = mi ϵ a b C B C , y he tratado de probar esto de la siguiente manera:

{ metro r ˙ a , metro r ˙ b } = { pag a mi A a ( r ) , pag b mi A b ( r ) }                                                                                                       = { pag a , pag b } mi { pag a , A b ( r ) } + mi { pag b , A a ( r ) } + mi 2 { A a ( r ) , A b ( r ) }

Ahora, el primer y el último término aquí son cero, por lo que esto se simplifica a:

{ metro r ˙ a , metro r ˙ b } = mi A b ( r ) r a mi A a ( r ) r b

usando el hecho de que { pag a , F ( r ) } = F ( r ) r a . No estoy muy seguro de adónde ir desde aquí, supongo que se necesita algún tipo de manipulación con el símbolo de Levi-Civita. Gracias de antemano.

¿Alguien puede explicar por qué esto está fuera de tema? @ jl2 "pregunta [ed] sobre un concepto de física específico" y "muestra [ed] algún esfuerzo para resolver el problema", ¿no? Creo que llegó al 90% del camino y solo necesitaba algunos consejos sobre el símbolo de Levi-Cevita, como dijo. Supongo que podría haberme detenido unos pasos en mi respuesta, ¿dejándolo terminarla? Aún así, eso sería con respecto a mi respuesta, no a su pregunta.
Lo siento realmente confundido por esto. No veo cómo la pregunta no es específica, o cómo no he mostrado ningún esfuerzo para llegar a una solución. ¿Hay alguna manera de que pueda recuperar la respuesta? Nada en el Centro de ayuda parece indicar que hay algún problema con esto.

Respuestas (1)

Voy a utilizar la notación de suma de Einstein en todo momento.

Ya casi estás ahí. Solo necesitas usar

B = × A
o equivalente,
B k = r i A j ϵ i j k = 1 2 ( r i A j ϵ i j k + r j A i ϵ j i k ) = 1 2 ( r i A j r j A i ) ϵ i j k
Allí usé la antisimetría de ϵ i j k bajo el intercambio de cualquiera de los dos índices.

Ahora usando la primera ecuación aquí ,

B k ϵ k r s = 1 2 ( r i A j r j A i ) ϵ k i j ϵ k r s = 1 2 ( r i A j r j A i ) ( d i r d j s d i s d j r ) = 1 2 ( r r A s r s A r ) 1 2 ( r s A r r r A s ) = r r A s r s A r